Đăng ký Đăng nhập
Trang chủ Về đa thức khả quy trên zp nhưng bất khả quy trên q...

Tài liệu Về đa thức khả quy trên zp nhưng bất khả quy trên q

.PDF
42
3
112

Mô tả:

.. ĐẠI HỌC THÁI NGUYÊN TRƯỜNG ĐẠI HỌC KHOA HỌC NGUYỄN VĂN LẬP VỀ ĐA THỨC KHẢ QUY TRÊN Z p NHƯNG BẤT KHẢ QUY TRÊN Q LUẬN VĂN THẠC SĨ TOÁN HỌC Thái Nguyên - 2015 ĐẠI HỌC THÁI NGUYÊN TRƯỜNG ĐẠI HỌC KHOA HỌC NGUYỄN VĂN LẬP VỀ ĐA THỨC KHẢ QUY TRÊN Z p NHƯNG BẤT KHẢ QUY TRÊN Q Chuyên ngành: Phương pháp Toán sơ cấp Mã số: 60 46 01 13 LUẬN VĂN THẠC SĨ TOÁN HỌC NGƯỜI HƯỚNG DẪN KHOA HỌC GS.TS. LÊ THỊ THANH NHÀN Thái Nguyên - 2015 i Mục lục Mục lục i Lời cảm ơn ii Danh sách ký hiệu iii Mở đầu 1 1 Đa thức bất khả quy 3 1.1 Định nghĩa, ví dụ và tính chất . . . . . . . . . . . . . . . . . 3 1.2 Đa thức bất khả quy của một phần tử . . . . . . . . . . . . . 4 2 Đa thức khả quy trên Zp nhưng bất khả quy trên Q 9 2.1 Một số tiêu chuẩn bất khả quy trên Q . . . . . . . . . . . . . 9 2.2 Mối quan hệ giữa tính bất khả quy trên Q và trên Zp . . . . . 13 2.3 Tính bất khả quy của đa thức bậc bốn trùng phương . . . . . 18 √ √ Tính bất khả quy trên Zp của đa thức bất khả quy của a + b 31 2.4 Kết luận 36 Tài liệu tham khảo 37 ii Lời cảm ơn Luận văn này được hoàn thành tại trường Đại học Khoa học - Đại học Thái Nguyên. Tác giả xin bày tỏ lòng biết ơn sâu sắc với GS. TS. Lê Thị Thanh Nhàn, đã trực tiếp hướng dẫn tận tình và động viên tác giả trong suốt thời gian nghiên cứu vừa qua. Tác giả xin chân thành cảm ơn các Thầy cô thuộc Khoa Toán - Tin, trường Đại học Khoa học và GS.TSKH Hà Huy Khoái, GS.TSKH Nguyễn Văn Mậu, PGS.TS Đàm Văn Nhỉ đã giảng dạy, trang bị cho chúng em những kiến thức cần thiết. Xin chân thành cảm ơn Phòng Đào tạo, trường Đại học Khoa học đã tạo điều kiện thuận lợi, động viên, khuyến khích tác giả trong suốt quá trình học tập. Tác giả cũng xin bày tỏ lòng biết ơn sâu sắc tới gia đình và người thân luôn khuyến khích, động viên tác giả trong suốt quá trình học tập và làm luận văn. Thái Nguyên, 2015 Nguyễn Văn Lập Học viên Cao học Toán K7D, Trường ĐH Khoa học - ĐH Thái Nguyên iii Danh sách ký hiệu K là một trường. K[x] là vành đa thức một biến x trên K Z là vành các số nguyên Q là trường các số hữu tỷ R là trường các số thực C là trường các số phức 1 Mở đầu Cho K là một trường và f (x) ∈ K[x] là một đa thức có bậc dương. Ta nói f (x) là bất khả quy nếu f (x) không là tích của hai đa thức có bậc thấp hơn. Ngược lại, nếu f (x) là tích của hai đa thức có bậc thấp hơn thì ta nói f (x) là khả quy trên K. Trong vành đa thức trên một trường, đa thức bất khả quy đóng vai trò quan trọng tương tự như vai trò của số nguyên tố trong vành số nguyên. Vì thế, rất nhiều nhà toán học đã quan tâm nghiên cứu tính bất khả quy của đa thức. Cho f (x) là một đa thức với hệ số nguyên. Để xét tính bất khả quy của f (x) trên Q, người ta thường sử dụng phương pháp rút gọn theo môđun một số nguyên tố. Cụ thể, nếu tồn tại một số nguyên tố p sao cho khi chuyển qua vành Zp [x], đa thức f (x) có bậc không đổi và là bất khả quy trên trường Zp , thì f (x) là bất khả quy trên Q. Một câu hỏi tự nhiên được đặt ra. Giả sử f (x) là khả quy trên trường Zp với mọi số nguyên tố p. Liệu rằng f (x) cũng khả quy trên Q? Người đầu tiên đưa ra câu trả lời phủ định cho câu hỏi trên là nhà toán học nổi tiếng David Hilbert. Một trong những đa thức như thế là x4 − 10x2 + 1, nó bất khả quy trên Q nhưng khả quy trên mọi trường Zp . Mục đích của luận văn là trình bày lại hai bài báo gần đây về lớp đa thức bất khả quy trên Q nhưng khả quy trên mọi trường Zp . 1. E. Driver, P. A. Leonard, K. S. Williams, Irreducible quartic polynomials with factorization modulo p, The Amer. Math. Monthly, 112 (2005), 2 876-890. 2. Kelly J. Pearson and Tan Zhang, Reducible over any finite field but irreducible over Q, Inter. Math. Forum, 33 (2008), 1607-1610. Ngoài phần mở đầu và tài liệu tham khảo, luận văn gồm 2 chương với nội dung chính như sau. Chương 1 trình bày một số khái niệm cơ bản về đa thức bất khả quy. Chương 2 là nội dung chính của luận văn, chương này trình bày một số tiêu chuẩn bất khả quy trên Q, mối quan hệ giữa tính bất khả quy trên Q và trên Zp , tính bất khả quy của đa thức bậc bốn trùng phương và tính bất khả √ √ quy trên Zp của đa thức bất khả quy của a + b. Thái Nguyên, ngày 20 tháng 11 năm 2015 Nguyễn Văn Lập Email: [email protected] 3 Chương 1 Đa thức bất khả quy Trong suốt chương này, luôn giả thiết K là một trường và K[x] là vành đa thức một biến x với hệ số trong K. 1.1 Định nghĩa, ví dụ và tính chất Định nghĩa 1.1.1. Đa thức f (x) ∈ K[x] được gọi là bất khả quy trên K nếu deg f (x) > 0 và f (x) không là tích của hai đa thức có bậc thấp hơn. Nhận xét: Tính chất bất khả quy phụ thuộc vào vành cơ sở. Chẳng hạn, đa thức x2 − 7 là bất khả quy trên Q nhưng không bất khả quy trên R; đa thức x2 + 3 là bất khả quy trên R nhưng không bất khả quy trên C. Bổ đề 1.1.2. Đa thức f (x) ∈ K[x] là bất khả quy trên K nếu và chỉ nếu f (x + a) là bất khả quy với mọi a ∈ K. Chứng minh. Cho a ∈ K, với mỗi h(x) ∈ K[x] đặt h1 (x) = h(x − a). Chú ý rằng deg h1 (x) = deg h(x). Vì thế f (x + a) = k(x)g(x) là phân tích của f (x + a) thành tích hai đa thức có bậc thấp hơn khi và chỉ khi f (x) = k1 (x)g1 (x) là phân tích của hai đa thức có bậc thấp hơn. Vì vậy f (x) bất khả quy khi và chỉ khi f (x + a) bất khả quy. Cho T là một trường chứa trường K. Nhắc lại rằng một phần tử a ∈ T được gọi là nghiệm của đa thức f (x) ∈ K[x] nếu f (a) = 0. 4 Kết quả sau đây thường được gọi là Định lí Bezout bé. Bổ đề 1.1.3. Phần tử a ∈ K là nghiệm của đa thức f (x) ∈ K[x] nếu và chỉ nếu tồn tại đa thức g(x) ∈ K[x] sao cho f (x) = (x − a)g(x). Bổ đề 1.1.4. Trên một trường K các phát biểu sau là đúng. i) Đa thức bậc nhất luôn bất khả quy; ii) Đa thức bậc 2 và bậc 3 là bất khả quy nếu và chỉ nếu nó không có nghiệm trong K. Chứng minh. i) Rõ ràng đa thức bậc nhất không thể là tích của hai đa thức bậc thấp hơn, do đó đa thức bậc nhất là bất khả quy. ii) Giả sử f (x) có nghiệm a ∈ K. Vì deg f (x) > 1 nên theo Bổ đề 1.1.3 ta có f (x) = (x − a)g(x) trong đó g(x) ∈ K[x] có bậc dương và deg g(x) = deg f (x) − 1. Do đó f (x) khả quy. Giả sử f (x) khả quy trên K, tức là f (x) = g(x)h(x) với deg g(x) < deg f (x) và deg h(x) < deg f (x). Vì f (x) có bậc 2 hoặc bậc 3 nên một trong hai đa thức g(x) hoặc h(x) là bậc nhất. Mà đa thức bậc nhất luôn có nghiệm trên trường đó. Vì vậy f (x) có nghiệm trong K. Chú ý rằng phát biểu (ii) của Bổ đề 1.1.4 là không đúng trong trường hợp đa thức có bậc lớn hơn 3. Cụ thể nếu đa thức bậc lớn hơn 3 có nghiệm trong K thì khả quy. Tuy nhiên tồn tại những đa thức không có nghiệm trong K nhưng vẫn khả quy. Ví dụ đa thức x4 + 5x2 + 6 = (x2 + 2)(x2 + 3) không có nghiệm trong R nhưng vẫn khả quy trên R. 1.2 Đa thức bất khả quy của một phần tử Trước hết chúng ta nhắc lại khái niệm phần tử đại số, phần tử siêu việt. 5 Định nghĩa 1.2.1. Cho F là một trường chứa trường K và α ∈ F . Ta nói rằng α là phần tử đại số trên K nếu α là nghiệm của đa thức khác 0 với hệ số trên K. Nếu α không đại số trên K thì ta nói α là phần tử siêu việt trên K. Đặc biệt, nếu α ∈ C là đại số (siêu việt) trên Q thì α được gọi là số đại số (số siêu việt) trên K. Ví dụ 1.2.2. Mọi số phức α = a + bi đều là đại số trên trường số thực R vì α √ là nghiệm của đa thức x2 − 2ax + a2 + b2 ∈ R[x]. Số thực 3 là số đại số vì nó là nghiệm của đa thức x2 − 3 ∈ Q[x]. Người ta đã chứng minh rằng số π (tỷ số giữa chu vi và bán kính của một đường tròn) không là nghiệm của bất cứ đa thức khác 0 nào trong Q[x]. Vì thế π là số siêu việt. Đa thức f (x) ∈ K[x] được gọi là có dạng chuẩn nếu hệ số cao nhất của nó bằng 1. Mệnh đề 1.2.3. Cho F là một trường chứa K và a ∈ F là phần tử đại số trên K. Khi đó, tồn tại duy nhất một đa thức dạng chuẩn p(x) ∈ K[x] bất khả quy nhận a làm nghiệm. Hơn nữa nếu g(x) ∈ K[x] nhận a làm nghiệm thì g(x) là bội của p(x). Chứng minh. Vì a là một phần tử đại số trên K nên tồn tại f (x) ∈ K[x] là đa thức khác 0 có bậc bé nhất nhận a làm nghiệm. Đặt p(x) = b−1 f (x) trong đó b là hệ số cao nhất của f (x). Khi đó p(x) ∈ K[x] là đa thức dạng chuẩn có bậc bé nhất nhận a làm nghiệm. Rõ ràng deg p(x) > 0. Nếu p(x) khả quy thì p(x) là tích của hai đa thức trong K[x] với bậc bé hơn p(x) và một trong hai đa thức này nhận a làm nghiệm, điều này mâu thuẫn với cách chọn p(x). Do đó f (x) bất khả quy. Giả sử g(x) ∈ K[x] nhận a làm nghiệm. Nếu g(x) không chia hết cho p(x) thì gcd(g(x), p(x)) = 1 vì p(x) bất khả quy. Do đó tồn tại q(x), h(x) ∈ K[x] sao cho 1 = p(x).q(x)+g(x).h(x). Thay x = a ta được 1 = p(a).q(a)+ 6 g(a).h(a) hay 1 = 0, điều này vô lý. Vậy g(x) chia hết cho p(x). Giả sử r(x) ∈ K[x] cũng là đa thức bất khả quy dạng chuẩn nhận a làm nghiệm. Theo chứng minh trên r(x) là bội của p(x), suy ra r(x) = p(x).k(x), k(x) ∈ K[x] vì r(x) bất khả quy nên k(x) = c với 0 6= c ∈ K. Do đó r(x) = c.p(x). Nhưng r(x), p(x) đều là đa thức dạng chuẩn nên c = 1. Vì thế p(x) = r(x). Từ mệnh đề trên, ta có khái niệm sau. Định nghĩa 1.2.4. Cho a là phần tử đại số trên K. Đa thức p(x) ∈ K[x] bất khả quy dạng chuẩn nhận a làm nghiệm được gọi là đa thức bất khả quy của a. Chẳng hạn đa thức p(x) = x3 − 5 ∈ Q(x) là bất khả quy vì p(x) có bậc 3 và không có nghiệm hữu tỷ. Do đó p(x) là đa thức bất khả quy của phần tử √ 3 5. Đa thức p(x) = x2 + 4 ∈ R(x) là bất khả quy vì p(x) có bậc 2 và không có nghiệm thực, do đó p(x) là đa thức bất khả quy của phần tử phức 2i. Bổ đề 1.2.5. Nếu phân số tối giản r/s là nghiệm hữu tỷ của đa thức với hệ số nguyên f (x) = an xn + . . . + a1 x + a0 thì r là ước của a0 và s là ước của an . Đặc biệt, nếu an = ±1 thì mọi nghiệm hữu tỷ của f (x) đều là nghiệm nguyên. Chứng minh. Giả sử r s là nghiệm của f (x) trong đó r, s ∈ Z và (r, s) = 1. Khi đó, ta có rn 0 = f (r/s) = an s + an−1 n rn−1 r + · · · + a 1 + a0 . sn−1 s Suy ra 0 = an rn + an−1 rn−1 s + · · · + a1 rsn−1 + a0 sn . Do đó an rn = −(an−1 rn−1 s + · · · + a1 rsn−1 + a0 sn ). (1) 7 Rõ ràng vế phải của đẳng thức này là bội của s. Vì thế vế trái an rn cũng là bội của s mà (r, s) = 1 nên (rn , s) = 1. Do vậy an là bội của s hay s là ước của an . Bây giờ ta chứng minh r là ước của a0 . Cũng từ (1) ta có a0 sn = −(an rn + an−1 rn−1 s + · · · + a1 rsn−1 ). Vế phải của đẳng thức này là bội của r. Vì thế vế trái a0 sn cũng là bội của r mà (r, s) = 1 suy ra (r, sn ) = 1. Do vậy a0 là bội của r hay r là ước của a0 . Việc tìm đa thức bất khả quy của một phần tử đại số nhìn chung là rất khó khăn. Trong một số trường hợp đặc biệt, chúng ta có thể sử dụng tiêu chuẩn nghiệm hữu tỷ trong Bổ đề 1.2.5 để xác định đa thức bất khả quy của một phần tử đại số trên Q. Ví dụ 1.2.6. Tìm đa thức bất khả quy trên Q của phần tử Giải. Đặt α = √ 3+ √ √ 3+ √ 5. √ √ 5. Khi đó α2 = 8 + 2 15 hay α2 − 8 = 2 15. Do đó α4 − 16α2 + 4 = 0, vì thế α là nghiệm của đa thức f (x) = x4 − 16x2 + 4 ∈ Q[x]. Ta chứng minh f (x) bất khả quy trên Q. Theo Bổ đề 1.2.5 thì nghiệm nếu có của f (x) là ước của 4 nhưng ±4; ±2; ±1 không thỏa mãn. Do vậy f (x) không có nghiệm hữu tỷ. Vì thế nó không là tích của đa thức bậc nhất và đa thức bậc 3. Giả sử f (x) khả quy. Khi đó x4 − 16x2 + 4 = (x2 + ax + b)(x2 + cx + d) (với a, b, c, d là hữu tỷ) hay x4 − 16x2 + 4 = x4 + (a + c)x3 + (b + d + ac)x2 + 8 (ad + bc)x + bd. Đồng nhất hệ số ở hai vế ta được hệ    a+c =0 (1)      b + d + ac = −16 (2)   ad + bc      bd =4 (3) = 4. (4) Từ (1) suy ra c = −a thế vào (3) ta được ad − ba = 0 hay a(d − b) = 0. Do đó a = 0 hoặc d = b. Nếu a = 0 thay vào (2) và (4) ta được   b + d = −16  bd = 4. Suy ra b2 + 16b + 4 = 0. Rõ ràng b ∈ / Q, vô lý. Nếu d = b thay vào (4) ta được b = d = ±2. Với b = d = −2 thay vào hệ (1), (2) ta được   a+c =0  ac = −12. Suy ra a2 = 12, a ∈ / Q, vô lí. Với b = d = 2 thay vào hệ (1), (2) ta được   a+c =0  ac = −20. Suy ra a2 = 20, a ∈ / Q, vô lí. Vì vậy f (x) = x4 − 16x2 + 4 là đa thức bất khả √ √ quy của 3 + 5. 9 Chương 2 Đa thức khả quy trên Zp nhưng bất khả quy trên Q 2.1 Một số tiêu chuẩn bất khả quy trên Q Định lí cơ bản của đại số phát biểu rằng mọi đa thức bậc dương với hệ số phức đều có ít nhất một nghiệm phức. Vì thế nếu f (x) ∈ C[x] là đa thức bậc n thì f (x) phân tích được thành tích của n đa thức tuyến tính (tức là đa thức bậc 1 ) với hệ số phức. Từ đây ta dễ dàng suy ra được mỗi đa thức f (x) ∈ R[x] đều phân tích được thành tích của các đa thức bậc 1 hoặc bậc hai với hệ số thực. Do đó các đa thức bất khả quy trên C là và chỉ là các đa thức bậc nhất; các đa thức bất khả quy trên R chỉ là các đa thức bậc nhất hoặc đa thức bậc hai với biệt thức âm (tức là đa thức bậc hai không có nghiệm thực). Tuy nhiên, việc xác định các đa thức bất khả quy trên Q đến nay vẫn là bài toán lớn, đang thách thức các nhà toán học trên thế giới. Mục tiêu của tiết này là trình bày một số phương pháp xét tính bất khả quy của đa thức trên Q như phương pháp tìm nghiệm hữu tỷ, phương pháp dùng Bổ đề Gauss, phương pháp dùng tiêu chuẩn Eisenstein. Giả sử f (x) ∈ Q[x]. Chú ý rằng f (x) bất khả quy trên Q khi và chỉ khi af (x) bất khả quy trên Q, trong đó a là mẫu chung nhỏ nhất của các hệ số của f (x). Khi đó af (x) ∈ Z[x]. Do đó, ta chỉ cần xét tính bất khả quy trên Q cho các đa thức với hệ số nguyên. Từ nay đến hết mục này, ta luôn giả thiết 10 f (x) = an xn + an−1 xn−1 + · · · + a0 ∈ Z[x], trong đó an 6= 0, n > 0. Nhận xét rằng đa thức bậc 1 thì bất khả quy trên Q, còn đa thức bậc lớn hơn 1 và có nghiệm hữu tỷ thì khả quy trên Q. Vì vậy, trong nhiều trường hợp ta có thể tìm nghiệm hữu tỷ để xét tính bất khả quy của f (x) trên Q. Sau đây, chúng ta lấy một số ví dụ minh họa. Ví dụ 2.1.1. a) Đa thức f (x) = 3x3 − 10x2 − 27x + 10 khả quy trên Q. b) Đa thức g(x) = 27x3 + 99x2 − 12x + 13 là bất khả quy trên Q. Chứng minh. a) Giả sử phân số tối giản r/s là nghiệm của f (x). Theo Bổ đề 1.2.5 thì r là ước của 10 và s là ước của 3. Suy ra r ∈ {±1, ±2, ±5, ±10} và s ∈ {±1, ±3}. Suy ra ( r/s ∈ ) 2 5 10 ± 1, ±2, ±5, ±10, ± , ± , ± , ± 3 3 3 3 1 1 Thử lại, ta thấy −2, 5, là nghiệm của f (x). Vậy f (x) khả quy trên Q. 3 b) Xét phương trình g(x) = 0 hay 27x3 + 99x2 − 12x + 13 = 0. Đặt y = 3x ta được y 3 + 11y 2 − 4y + 13 = 0. Xét h(y) = y 3 + 11y 2 − 4y + 13 nếu h(y) có nghiệm hữu tỷ thì nghiệm đó phải là số nguyên (do h(y) là đa thức dạng chuẩn). Thử lại thấy ±1, ±13 không là nghiệm của phương trình h(y) = 0 hay phương trình h(y) = 0 không có nghiệm hữu tỷ. Theo Bổ đề 1.2.5, h(y) không có nghiệm hữu tỷ. Vì vậy g(x) = 0 không có nghiệm hữu tỷ. Do đó g(x) bất khả quy trên Q. Tiếp theo chúng ta trình bày phương pháp dùng Bổ đề Gauss. Nhắc lại rằng một đa thức với hệ số nguyên được gọi là đa thức nguyên bản nếu các hệ số của nó nguyên tố cùng nhau. Chú ý rằng tích của hai đa thức nguyên bản là một đa thức nguyên bản. 11 Định lí 2.1.2 (Bổ đề Gauss). Cho p(x) ∈ Z[x]. Giả sử p(x) = g(x)f (x) với g(x), f (x) ∈ Q[x]. Khi đó, tồn tại các đa thức g∗ (x), f∗ (x) ∈ Z[x] sao cho deg g(x) = deg g∗ (x), deg f (x) = deg f∗ (x) và p(x) = g∗ (x)f∗ (x). Đặc biệt, nếu p(x) là khả quy trên Q thì nó được phân tích thành tích của hai đa thức với hệ số nguyên có bậc thấp hơn. Chứng minh. Viết f (x) = af1 (x) và g(x) = bg1 (x) trong đó a, b ∈ Q và f1 (x), g1 (x) ∈ Z[x] là các đa thức nguyên bản. Suy ra f1 (x)g1 (x) là đa thức nguyên bản. Rõ ràng p(x) = abf1 (x)g1 (x) ∈ Z[x]. Ta chứng minh ab ∈ Z. Thật vậy, giả sử ab 6∈ Z. Khi đó ab = r/s với r/s là phân số tối giản và s > 1. Viết f1 (x)g1 (x) = an xn + · · · + a1 x + a0 . Vì f1 (x)g1 (x) là nguyên bản nên gcd(an , an−1 , . . . , a0 ) = 1. Vì p(x) ∈ Z[x] nên ta có ran s ,..., ra1 ra0 , ∈ Z. s s Suy ra s là ước chung của an , . . . , a1 , a0 điều này vô lí. Vậy ab ∈ Z. Đặt f∗ (x) = abf1 (x) và g∗ (x) = g1 (x). Khi đó p(x) = f∗ (x)g∗ (x) với f∗ (x)g∗ (x) ∈ Z[x] và deg f (x) = deg f∗ (x) và deg g(x) = deg g∗ (x). Dựa vào Bổ đề Gauss, chúng ta xét tính bất khả quy của đa thức thông qua ví dụ sau. Ví dụ 2.1.3. Đa thức f (x) = x4 + 2x3 + x − 5 bất khả quy trên Q . Chứng minh. Nếu f (x) có nghiệm hữu tỷ thì theo Bổ đề 1.2.5 nghiệm đó thuộc {±1, ±5}. Nhưng ±1, ±5 không thỏa mãn phương trình f (x) = 0. Do vậy f (x) không có nghiệm hữu tỷ. Vì thế f (x) không là tích của một đa thức bậc nhất và một đa thức bậc ba. Giả sử f (x) khả quy. Theo Bổ đề Gauss, f (x) có sự phân tích f (x) = g(x)h(x) trong đó g(x), h(x) ∈ Z[x] có bậc 2 và có hệ số cao nhất bằng 1. 12 Viết g(x) = x2 + ax + b và h(x) = x2 + cx + d với a, b, c, d ∈ Z. Khi đó f (x) = x4 + (a + c)x3 + (b + d + ac)x2 + (ad + bc)x + bd. Đồng nhất các hệ số, ta được    a+c      b + d + ac   bc + ad      bd =2 =0 =1 = −5. Vì vai trò của b, d là như nhau nên không mất tính tổng quát ta có thể giả thiết b = 1, d = −5 hoặc b = −1, d = 5. Nếu b = 1, d = −5 thì a + c = 2, ac = 4. Suy ra a(2 − a) = 4 hay a2 − 2a − 4 = 0, khi đó a 6∈ Z, vô lí. Tương tự, nếu b = −1 và d = 5 thì a + c = 2 = 0, ac = −4, dẫn đến phương trình a2 − 2a − 4 = 0, khi đó a 6∈ Z, vô lí. Như vậy, f (x) bất khả quy. Tiếp theo, ta trình bày tiêu chuẩn Eisenstein để xét tính bất khả quy của đa thức trên Q. Định lí 2.1.4 (Tiêu chuẩn Eisenstein). Cho đa thức f = an xn + · · · + a1 x + a0 ∈ Z[x]. Giả sử tồn tại một số nguyên tố p thỏa mãn: i) p không là ước của hệ số cao nhất an ; ii) p là ước của các hệ số a0 , a1 , . . . , an−1 ; iii) p2 không là ước của hệ số tự do a0 . Khi đó f (x) là bất khả quy trên Q. Chứng minh. Giả sử f (x) khả quy trên Q. Theo Bổ đề Gauss, tồn tại phân tích f (x) = g(x)h(x), trong đó g(x) = bm xm + · · · + b1 x + b0 ∈ Z[x] và 13 h(x) = ck xk + · · · + c1 x + c0 ∈ Z[x] với deg g(x) = m, deg h(x) = k và m, k < n. Do p là ước của a0 = b0 c0 nên p|b0 hoặc p|c0 . Lại do p2 không là ước của a0 nên trong hai số b0 và c0 có một và chỉ một số chia hết cho p. Giả thiết p|c0 . Khi đó b0 không chia hết cho p. Vì an = bm ck và an không chia hết cho p nên bm và ck đều không chia hết cho p. Do đó, tồn tại số r bé nhất sao cho cr không là bội của p. Ta có ar = b0 cr + (b1 cr−1 + b2 cr−2 + · · · + br c0 ). Vì r ≤ k < n nên p|ar . Theo cách chọn r ta có p|(b1 cr−1 + b2 cr−2 + · · · + br c0 ). Suy ra p|b0 cr , điều này là vô lí vì cả hai số b0 và cr đều không là bội của p. Vậy f (x) là bất khả quy trên Q. Ví dụ 2.1.5. i) Đa thức x5 + 3x4 + 48 là bất khả quy trên Q theo tiêu chuẩn Eisenstein với p = 3. ii) Đa thức 7x10 + 6x5 − 12x + 10 là bất khả quy trên Q theo tiêu chuẩn Eisenstein với p = 2. iii) Đa thức 6x2015 + 5x10 − 25x2 + 35 là bất khả quy trên Q theo tiêu chuẩn Eisenstein với p = 5. 2.2 Mối quan hệ giữa tính bất khả quy trên Q và trên Zp Trong phần này, chúng ta trình bày phương pháp rút gọn theo môđun một số nguyên tố để xét tính bất khả quy trên Q. Chúng ta sẽ chỉ ra rằng với mỗi đa thức f (x) ∈ Z[x], nếu tồn tại một số nguyên tố p sao cho khi chuyển f (x) vào vành Zp [x] ta được đa thức có cùng bậc và bất khả quy trên Zp thì f (x) là bất khả quy trên Q. Tiếp theo, chúng ta trình bày một số ví dụ xét tính bất khả quy trên Q bằng phương pháp rút gọn theo môđun một số nguyên tố. Cuối cùng chúng ta chỉ ra một ví dụ chứng tỏ rằng có những đa thức bất khả quy trên Q nhưng khả quy trên mọi trường Zp . Để thuận tiện, chúng ta dùng các số nguyên a, b, ... để kí hiệu cho các phần 14 tử của Zp với chú ý rằng a = b trong Zp nếu và chỉ nếu a − b chia hết cho p trong vành Z. Với mỗi đa thức f (x) = an xn + an−1 xn−1 + · · · + a1 x1 + a0 ∈ Z[x] và mỗi số nguyên tố p, ta đặt f (x) = an xn + · · · + a1 x + a0 ∈ Zp [x]. Định lý sau đây cho ta một công cụ rất mạnh để xét tính bất khả quy trên Q của đa thức với hệ số nguyên. Định lí 2.2.1. Nếu tồn tại một số nguyên tố p sao cho deg f (x) = deg f (x) và f (x) bất khả quy trên Zp thì f (x) bất khả quy trên Q. Chứng minh. Vì f (x) bất khả quy trên Zp nên deg f (x) > 0. Lại do deg f (x) = deg f (x) nên deg f (x) > 0. Giả sử f (x) khả quy trên Q, theo Bổ đề Gauss f (x) có phân tích f (x) = g(x).h(x) với g(x), h(x) ∈ Z[x] và g(x), h(x) có bậc nhỏ hơn bậc của f (x). Chú ý rằng f (x) = g(x).h(x), do đó deg f (x) = deg g(x) + deg h(x). Mà deg g(x) ≥ deg g(x) và deg h(x) ≥ deg h(x), mặt khác deg f (x) = deg f (x) nên deg g(x) = deg g(x) và deg h(x) = deg h(x). Do đó f (x) phân tích được thành tích của hai đa thức có bậc thấp hơn, hay f (x) khả quy. Điều này mâu thuẫn với f (x) bất khả quy. Vậy định lý được chứng minh. Ví dụ 2.2.2. Các đa thức sau là bất khả quy trên Q (i) f (x) = 14x2 + 9x + 7. (ii) g(x) = 5x3 + 6x2 − x + 13. (iii) h(x) = 6x4 − 10x3 + 11x2 − 7x + 2. Chứng minh. (i) Vì f (x) = 2x2 −x+1 ∈ Z3 [x] không có nghiệm trong Z3 và deg f (x) = 2 nên f (x) bất khả quy trên Z3 . Rõ ràng deg f (x) = deg f (x) = 2 , nên f (x) bất khả quy trên Q theo Định lý 2.2.1. (ii) Vì g(x) = x3 +x+1 ∈ Z2 [x] không có nghiệm trong Z2 và deg g(x) = 3 nên g(x) bất khả quy trên Z3 . Rõ ràng deg g(x) = deg g(x) = 3. Theo Định lý 2.2.1 ta suy ra g(x) là bất khả quy trên Q. 15 (iii) Vì h(x) = x4 + x2 + 3x + 2 ∈ Z5 [x] không có nghiệm trong Z5 , do đó h(x) không thể là tích của một đa thức bậc nhất và một đa thức bậc ba. Giả sử h(x) khả quy trên Z5 , khi đó h(x) là tích của hai đa thức bậc hai h(x) = (x2 + ax + b)(x2 + cx + d) với a, b, c, d ∈ Z5 hay h(x) = x4 + (a + c)x3 + (b + d + ac)x2 + (ad + bc)x + bd. Đồng nhất hệ số, ta được các đẳng thức sau trong vành Z5    a+c =0      b + d + ac = 1   ad + bc      bd =3 = 2. Từ bd = 2 và b, d ∈ Z5 và vai trò của b, d như nhau nên (b, d) bằng (1, 2) hoặc (3, 4). Nếu b = 1 và d = 2 thay vào hai phương trình đầu, ta được a + c = 0 và ac = 3. Không có a, c ∈ Z5 thỏa mãn. Nếu b = 3 và d = 4 thay vào ta được    a+c =0   ac =4     4a + 3c = 3. Không tìm được a thỏa mãn. Vì vậy h(x) bất khả quy trên Z5 mà deg h(x) = deg h(x). Do vậy h(x) bất khả quy trên Q theo Định lý 2.2.1. Như vậy, theo Định lí 2.2.1, nếu tồn tại một số nguyên tố p để f (x) bất khả quy trên Zp và f (x) có bậc không đổi khi chuyển vào Zp [x] thì f (x) là bất khả quy trên Q. Trong phần cuối của tiết này, chúng ta chỉ ra rằng điều ngược lại là không đúng. Dưới đây, chúng ta đưa ra một kết quả chỉ ra sự tồn tại một đa thức f (x) ∈ Z[x] bất khả quy trên Q nhưng khả quy trên mọi trường Zp .
- Xem thêm -

Tài liệu liên quan

Tài liệu xem nhiều nhất